Difference between revisions of "1950 AHSME Problems/Problem 36"

(Solution)
 
(4 intermediate revisions by 4 users not shown)
Line 1: Line 1:
 
==Problem==
 
==Problem==
  
A merchant buys goods at <math> 25\%</math> of the list price. He desires to mark the goods so that he can give a discount of <math> 20\%</math> on the marked price and still clear a profit of <math> 25\%</math> on the selling price. What percent of the list price must he mark the goods?
+
A merchant buys goods at <math> 25\%</math> off the list price. He desires to mark the goods so that he can give a discount of <math> 20\%</math> on the marked price and still clear a profit of <math> 25\%</math> on the selling price. What percent of the list price must he mark the goods?
  
 
<math>\textbf{(A)}\ 125\% \qquad
 
<math>\textbf{(A)}\ 125\% \qquad
Line 10: Line 10:
  
 
==Solution==
 
==Solution==
{{solution}}
+
Without loss of generality, we can set the list price equal to <math>100</math>. The merchant buys the goods for <math>100*.75=75</math>. Let <math>x</math> be the marked price.
 +
We then use the equation <math>0.8x-75=25</math> to solve for <math>x</math> and get a marked price of <math>\boxed{125}</math>.
 +
 
 +
<math>\text{Answer: }\boxed{\mathbf{(A)}}</math>
  
 
==See Also==
 
==See Also==
Line 16: Line 19:
  
 
[[Category:Introductory Algebra Problems]]
 
[[Category:Introductory Algebra Problems]]
 +
{{MAA Notice}}

Latest revision as of 21:41, 24 January 2018

Problem

A merchant buys goods at $25\%$ off the list price. He desires to mark the goods so that he can give a discount of $20\%$ on the marked price and still clear a profit of $25\%$ on the selling price. What percent of the list price must he mark the goods?

$\textbf{(A)}\ 125\% \qquad \textbf{(B)}\ 100\% \qquad \textbf{(C)}\ 120\% \qquad \textbf{(D)}\ 80\% \qquad \textbf{(E)}\ 75\%$

Solution

Without loss of generality, we can set the list price equal to $100$. The merchant buys the goods for $100*.75=75$. Let $x$ be the marked price. We then use the equation $0.8x-75=25$ to solve for $x$ and get a marked price of $\boxed{125}$.

$\text{Answer: }\boxed{\mathbf{(A)}}$

See Also

1950 AHSC (ProblemsAnswer KeyResources)
Preceded by
Problem 35
Followed by
Problem 37
1 2 3 4 5 6 7 8 9 10 11 12 13 14 15 16 17 18 19 20 21 22 23 24 25 26 27 28 29 30 31 32 33 34 35 36 37 38 39 40 41 42 43 44 45 46 47 48 49 50
All AHSME Problems and Solutions

The problems on this page are copyrighted by the Mathematical Association of America's American Mathematics Competitions. AMC logo.png